next up previous contents
suivant: Fonctions d'une variable réelle monter: Polynômes et fractions rationnelles précédent: Exercices   Table des matières

Solutions des exercices

Solution 1.1 : Les domaines de définition sont égaux à $ \mathbb{R}.$ Après simplification, il vient :

$\displaystyle f\left( x\right) =\frac{x^{8}-16}{x^{4}+4}=x^{4}-4$ et $\displaystyle g\left(
x\right) =10+2x^{2}\newline
$

donc $ f$ et $ g$ sont bien des fonctions polynômes.



Solution 1.2 : Par identification des coefficients, on trouve que $ a=2,$ $ b=1$ et $ c=0.$



Solution 1.3 : On trouve facilement
$ PQ=-6X^{5}-19X^{4}-X^{3}-4X^{2}-17X+21$
$ P^{2}
=\allowbreak9X^{4}+12X^{3}-14X^{2}-12X+9$
$ Q^{2}=\allowbreak
4X^{6}+20X^{5}+21X^{4}+18X^{3}+71X^{2}-14X+49$



Solution 1.4 : On développe le polynôme donné :

$\displaystyle X\left( X+a\right) \left( X+2a\right) \left( X+3a\right) +a^{4}
=\allowbreak X^{4}+6X^{3}a+11X^{2}a^{2}+6Xa^{3}+a^{4}
$

Si ce polynôme est le carré d'un polynôme unitaire, alors il existe $ \alpha$ et $ \beta$ réels tels que :

$\displaystyle \left( X^{2}+\alpha X+\beta\right) ^{2}=\allowbreak X^{4}+6X^{3}
a+11X^{2}a^{2}+6Xa^{3}+a^{4}
$

On développe le carré, puis on identifie les coefficients, d'où :

$\displaystyle X^{4}+2X^{3}\alpha+X^{2}\left( 2\beta+\alpha^{2}\right) +2\alpha
X\beta+\beta^{2}=\allowbreak X^{4}+6X^{3}a+11X^{2}a^{2}+6Xa^{3}+a^{4}
$

On trouve après calcul par exemple les valeurs $ \alpha=3a$ et $ \beta
=a^{2}.$ Le polynôme $ X^{2}+3aX+a^{2}$ répond donc à la question.



Solution 1.5 : On a $ P\left( 2\right) =0$ et $ P\left( -3\right) =0,$ d'où $ a=106$ et $ b=-204.$



Solution 1.6 : Pour tout $ x$ de $ \mathbb{R}
_{+},$ $ P\left( x\right) >0,$ donc les racines éventuelles appartiennent à $ \mathbb{R}_{-}^{\ast}.$



Solution 1.7 : $ P\left( \frac{1}{2}\right)
=P\left( -\frac{1}{2}\right) =0.$ Comme ces deux racines sont distinctes, on en déduit que $ P\left( x\right) $ se factorise par $ \left( x-\frac{1}{2}\right) \left( x+\frac{1}
{2}\right) .$



Solution 1.8 : $ P\left( 1\right) =0$ entraîne que la somme des coefficients est nulle. Seul le polynôme du cas a) est factorisable par $ \left( x-1\right) ,$ et on a :

$\displaystyle P\left( x\right) =8x^{5}-3x^{4}+6x^{2}-4x-7=\left( x-1\right) \left(
8x^{4}+5x^{3}+5x^{2}+11x+7\right)
$



Solution 1.9 : On vérifie que $ P\left(
1\right) =P^{\prime}\left( 1\right) =P^{\prime\prime}\left( 1\right) =0$ et que $ P^{\prime\prime\prime}\left( 1\right) \neq0.$ En effet :

$\displaystyle P\left( X\right)$ $\displaystyle =-2X^{4}+10X^{3}-18X^{2}+14X-4$ d'où $\displaystyle P\left( 1\right) =0$    
$\displaystyle P^{\prime}\left( X\right)$ $\displaystyle =-8X^{3}+30X^{2}-36X+14$ d'où $\displaystyle P^{\prime}\left( 1\right) =0$    
$\displaystyle P^{\prime\prime}\left( X\right)$ $\displaystyle =\allowbreak-24X^{2}+60X-36$ d'où $\displaystyle P^{\prime\prime}\left( 1\right) =0$    
$\displaystyle P^{\prime\prime\prime}\left( X\right)$ $\displaystyle =-48X+60$ d'où $\displaystyle P^{\prime\prime\prime}\left( 1\right) =12\neq0$    



Solution 1.10 : On obtient :

$\displaystyle 2X^{5}+X^{4}+2X^{3}-1=\left( \allowbreak2X^{3}+3X^{2}-X-10\right) \left(
X^{2}-X+3\right) +\left( 29-7X\right)
$



Solution 1.11 : On remarque que :

$\displaystyle \left( -x^{3}+4x^{2}+\frac{27}{7}x+\frac{37}{7}\right) \left( x+3\right)
+\left( x^{2}+x+1\right) \left( x^{2}-2x-\frac{104}{7}\right) =1
$

ce qui prouve d'après l'égalité de Bezout que ces deux polynômes sont premiers entre eux.



Solution 1.12 : On obtient :

PGCD$\displaystyle \left( P,Q\right)$ $\displaystyle =\left( X-1\right) ^{2}\left( X+1\right) ^{4}$    
PPCM$\displaystyle \left( P,Q\right)$ $\displaystyle =X^{3}\left( X-1\right) ^{3}\left( X+2\right) \left( X+1\right) ^{5}$    



Solution 1.14 : On obtient les résultats suivants :

$\displaystyle Q_{1}\left( X\right) =\frac{X+1}{\left( X-2\right) \left( X+3\right)
}=\frac{3}{5\left( X-2\right) }+\frac{2}{5\left( X+3\right) }
$

$\displaystyle Q_{2}\left( X\right) =\frac{1}{X^{3}\left( X+1\right) ^{2}}=\frac...
...}}-\frac{2}{X^{2}}+\frac{3}{X}-\frac{1}{\left( X+1\right) ^{2}}
-\frac{3}{X+1}
$

$\displaystyle Q_{3}\left( X\right) =\frac{X^{2}+X+1}{\left( X-1\right) ^{2}\lef...
...{4\left( X+1\right) ^{2}}-\frac{1}{4\left(
X+1\right) }+\allowbreak\frac{1}{X}
$

$\displaystyle Q_{4}\left( X\right) =\frac{2X^{4}+X^{2}-1}{X\left( X-1\right) ^{2}
}=2X+4-\frac{1}{X}+\frac{2}{\left( -1+X\right) ^{2}}+\frac{8}{-1+X}
$



Solution 1.15 : On trouve :

$\displaystyle f(x)=-\frac{x^3}{2}+\frac{3}{2}x $



Solution 1.16 : Le polynôme d'interpolation de Lagrange est défini par :

$\displaystyle P(x)=1A(x)+2B(x)+(-1)C(x)+5D(x) $

$\displaystyle A(x)=\frac{(x-2)(x-3)(x-4)}{(1-2)(1-3)(1-4)} $

Il reste à déterminer les autres polynômes...



Solution 1.18 : On étudie sur $ \mathbb{R}$ la fonction

$\displaystyle x\mapsto P\left( x\right) =x^{3}-4x^{2}-17x+60
$

et on montre l'existence des trois racines réelles à l'aide du théorème des valeurs intermédiaires. Soit $ a$ une de ces racines. D'après le texte, $ a+2$ est aussi une racine de $ P,$ d'où

\begin{displaymath}
\left\{
\begin{array}[c]{l}
P\left( a\right) =0\\
P\left( a...
...3}-4a^{2}-17a+60=0\\
a^{3}+2a^{2}-21a+18=0
\end{array}\right.
\end{displaymath}

Par soustraction, on trouve que

$\displaystyle -6a^{2}+4a+42=0,$ d'où $\displaystyle a=-\frac{7}{3}$ ou $\displaystyle a=3
$

Après vérification, seul $ a=3$ convient, d'où $ 5$ est la deuxième racine. Enfin, comme le produit des racines du polynôme est égal à $ -60,$ cela permet de trouver que $ -4$ est la dernière racine cherchée.



Solution 1.19 : On appelle $ n$ le degré de $ P$. Vérifier que l'égalité proposée impose que $ n=3 $. A l'aide d'un système d'équations, résoudre alors le problème posé. On démontrera que l'ensemble $ S
$ des solutions est égal à :

$\displaystyle S=\left\lbrace 0,\frac{4}{9}X^3 \right\rbrace $


next up previous contents
suivant: Fonctions d'une variable réelle monter: Polynômes et fractions rationnelles précédent: Exercices   Table des matières
Michel 2002-08-06